Consecutive Integers

This topic has expert replies
User avatar
Senior | Next Rank: 100 Posts
Posts: 48
Joined: Sun Jul 20, 2008 4:45 pm
Location: Tallahassee, FL
Thanked: 2 times

Consecutive Integers

by tlt2372 » Tue Sep 28, 2010 3:45 pm
If x is an integer, then x(x - 1)(x - k) must be evenly divisible by three when k is any of the following values EXCEPT

A) -4
B) -2
C) -1
D) 2
E) 5

OA: B

Does anyone know a quick way to solve this problem?
THanks!

Legendary Member
Posts: 1119
Joined: Fri May 07, 2010 8:50 am
Thanked: 29 times
Followed by:3 members

by diebeatsthegmat » Tue Sep 28, 2010 4:31 pm
tlt2372 wrote:If x is an integer, then x(x - 1)(x - k) must be evenly divisible by three when k is any of the following values EXCEPT

A) -4
B) -2
C) -1
D) 2
E) 5

OA: B

Does anyone know a quick way to solve this problem?
THanks!
the best solution is choose a number...
x=2 so k=-1 or 5 -4
x=8 so k=5,2,-1,-4
the answer should be B
thus the answer is

User avatar
Master | Next Rank: 500 Posts
Posts: 307
Joined: Sun Jul 11, 2010 7:52 pm
Thanked: 36 times
Followed by:1 members
GMAT Score:640

by limestone » Tue Sep 28, 2010 8:50 pm
Trick for this question:

5 and 2 is the same to 3:
5=3+2
Both 5 and 2 if be divided to 3, will give out the remainder 2. Then if 5 is the answer choice, 2 must be too.

-4 and -1 is the same to 3:
-4 = -3 + (-1)

B is odd, so B must be the correct choice.
"There is nothing either good or bad - but thinking makes it so" - Shakespeare.

User avatar
GMAT Instructor
Posts: 3650
Joined: Wed Jan 21, 2009 4:27 am
Location: India
Thanked: 267 times
Followed by:80 members
GMAT Score:760

by sanju09 » Tue Sep 28, 2010 10:58 pm
tlt2372 wrote:If x is an integer, then x(x - 1)(x - k) must be evenly divisible by three when k is any of the following values EXCEPT

A) -4
B) -2
C) -1
D) 2
E) 5

OA: B

Does anyone know a quick way to solve this problem?
THanks!
The product of three consecutive integers is always divisible by 3. Hence, k can initially take any of the values -1 and 2, and 3 thereafter in each direction, which allows -4 and 5, what is not allowed is [spoiler]-2, from the choices.


B
[/spoiler]
The mind is everything. What you think you become. -Lord Buddha



Sanjeev K Saxena
Quantitative Instructor
The Princeton Review - Manya Abroad
Lucknow-226001

www.manyagroup.com

User avatar
Master | Next Rank: 500 Posts
Posts: 261
Joined: Wed Mar 31, 2010 8:37 pm
Location: Varanasi
Thanked: 11 times
Followed by:3 members

by ankur.agrawal » Wed Sep 29, 2010 1:05 am
sanju09 wrote:
tlt2372 wrote:If x is an integer, then x(x - 1)(x - k) must be evenly divisible by three when k is any of the following values EXCEPT

A) -4
B) -2
C) -1
D) 2
E) 5

OA: B

Does anyone know a quick way to solve this problem?
THanks!
The product of three consecutive integers is always divisible by 3. Hence, k can initially take any of the values -1 and 2, and 3 thereafter in each direction, which allows -4 and 5, what is not allowed is [spoiler]-2, from the choices.


Hey Sanju Can u explain Why -4 & 5 is allowed?

User avatar
GMAT Instructor
Posts: 3650
Joined: Wed Jan 21, 2009 4:27 am
Location: India
Thanked: 267 times
Followed by:80 members
GMAT Score:760

by sanju09 » Wed Sep 29, 2010 1:12 am
ankur.agrawal wrote:
sanju09 wrote:
tlt2372 wrote:If x is an integer, then x(x - 1)(x - k) must be evenly divisible by three when k is any of the following values EXCEPT

A) -4
B) -2
C) -1
D) 2
E) 5

OA: B

Does anyone know a quick way to solve this problem?
THanks!
The product of three consecutive integers is always divisible by 3. Hence, k can initially take any of the values -1 and 2, and 3 thereafter in each direction, which allows -4 and 5, what is not allowed is [spoiler]-2, from the choices.


Hey Sanju Can u explain Why -4 & 5 is allowed?


If x is one of the three numbers whose product is divisible by 3, then using x - 3 or x + 3 in place of x, would still keep the product divisible by 3, no matter is x is divisible by 3 or not.
The mind is everything. What you think you become. -Lord Buddha



Sanjeev K Saxena
Quantitative Instructor
The Princeton Review - Manya Abroad
Lucknow-226001

www.manyagroup.com

User avatar
Master | Next Rank: 500 Posts
Posts: 261
Joined: Wed Mar 31, 2010 8:37 pm
Location: Varanasi
Thanked: 11 times
Followed by:3 members

by ankur.agrawal » Wed Sep 29, 2010 1:25 am
sanju09 wrote:
ankur.agrawal wrote:
sanju09 wrote:
tlt2372 wrote:If x is an integer, then x(x - 1)(x - k) must be evenly divisible by three when k is any of the following values EXCEPT

A) -4
B) -2
C) -1
D) 2
E) 5

OA: B

Does anyone know a quick way to solve this problem?
THanks!
The product of three consecutive integers is always divisible by 3. Hence, k can initially take any of the values -1 and 2, and 3 thereafter in each direction, which allows -4 and 5, what is not allowed is [spoiler]-2, from the choices.


Hey Sanju Can u explain Why -4 & 5 is allowed?


If x is one of the three numbers whose product is divisible by 3, then using x - 3 or x + 3 in place of x, would still keep the product divisible by 3, no matter is x is divisible by 3 or not.


Sorry but i couldnt grasp this concept. If we take k=-4 then

(x) (x-1) (x+4) ---- is this a series of consecutive integers.? What am i missing. Plz explain

User avatar
GMAT Instructor
Posts: 3650
Joined: Wed Jan 21, 2009 4:27 am
Location: India
Thanked: 267 times
Followed by:80 members
GMAT Score:760

by sanju09 » Wed Sep 29, 2010 3:29 am
ankur.agrawal wrote:
sanju09 wrote:
ankur.agrawal wrote:
sanju09 wrote:
tlt2372 wrote:If x is an integer, then x(x - 1)(x - k) must be evenly divisible by three when k is any of the following values EXCEPT

A) -4
B) -2
C) -1
D) 2
E) 5

OA: B

Does anyone know a quick way to solve this problem?
THanks!
The product of three consecutive integers is always divisible by 3. Hence, k can initially take any of the values -1 and 2, and 3 thereafter in each direction, which allows -4 and 5, what is not allowed is [spoiler]-2, from the choices.


Hey Sanju Can u explain Why -4 & 5 is allowed?


If x is one of the three numbers whose product is divisible by 3, then using x - 3 or x + 3 in place of x, would still keep the product divisible by 3, no matter if x is divisible by 3 or not.


Sorry but i couldnt grasp this concept. If we take k=-4 then

(x) (x-1) (x+4) ---- is this a series of consecutive integers.? What am i missing. Plz explain


x and x - 1 are two consecutive integers, one of which may or may not be a multiple of 3, and since the product of three consecutive integers is always divisible by 3, hence the nearest third integer to make sure the product is divisible by 3, could be either x + 1 or x - 2, that's why k can initially be taken as -1 or 2.

Now, if x and x - 1 are two consecutive integers, one of which is a multiple of 3, then x (x - 1) (x - k) will always be divisible by 3 for any value of k; but if it's not so, then it will be divisible by 3 for some specific values of k only. Because, then x - k has to be divisible by 3, and if so, then both x + 1 and x - 2 are divisible by 3. And if so, then 3 more than x + 1 and 3 less than x - 2 will also be divisible by 3. That is, x + 4 and x - 5 will also be divisible by 3, and hence all of the following products will also be divisible by 3

A. x (x - 1) (x + 4)

C. x (x - 1) (x + 1)

D. x (x - 1) (x - 2)

E. x (x - 1) (x - 5)


EXCEPT

B. x (x - 1) (x + 2)

It's not necessary for the three integers to be always consecutive for their product to be divisible by 3.
The mind is everything. What you think you become. -Lord Buddha



Sanjeev K Saxena
Quantitative Instructor
The Princeton Review - Manya Abroad
Lucknow-226001

www.manyagroup.com

User avatar
Master | Next Rank: 500 Posts
Posts: 261
Joined: Wed Mar 31, 2010 8:37 pm
Location: Varanasi
Thanked: 11 times
Followed by:3 members

by ankur.agrawal » Wed Sep 29, 2010 12:34 pm
sanju09 wrote:
ankur.agrawal wrote:
sanju09 wrote:
ankur.agrawal wrote:
sanju09 wrote:
tlt2372 wrote:If x is an integer, then x(x - 1)(x - k) must be evenly divisible by three when k is any of the following values EXCEPT

A) -4
B) -2
C) -1
D) 2
E) 5

OA: B

Does anyone know a quick way to solve this problem?
THanks!
The product of three consecutive integers is always divisible by 3. Hence, k can initially take any of the values -1 and 2, and 3 thereafter in each direction, which allows -4 and 5, what is not allowed is [spoiler]-2, from the choices.


Hey Sanju Can u explain Why -4 & 5 is allowed?


If x is one of the three numbers whose product is divisible by 3, then using x - 3 or x + 3 in place of x, would still keep the product divisible by 3, no matter if x is divisible by 3 or not.


Sorry but i couldnt grasp this concept. If we take k=-4 then

(x) (x-1) (x+4) ---- is this a series of consecutive integers.? What am i missing. Plz explain


x and x - 1 are two consecutive integers, one of which may or may not be a multiple of 3, and since the product of three consecutive integers is always divisible by 3, hence the nearest third integer to make sure the product is divisible by 3, could be either x + 1 or x - 2, that's why k can initially be taken as -1 or 2.

Now, if x and x - 1 are two consecutive integers, one of which is a multiple of 3, then x (x - 1) (x - k) will always be divisible by 3 for any value of k; but if it's not so, then it will be divisible by 3 for some specific values of k only. Because, then x - k has to be divisible by 3, and if so, then both x + 1 and x - 2 are divisible by 3. And if so, then 3 more than x + 1 and 3 less than x - 2 will also be divisible by 3. That is, x + 4 and x - 5 will also be divisible by 3, and hence all of the following products will also be divisible by 3

A. x (x - 1) (x + 4)

C. x (x - 1) (x + 1)

D. x (x - 1) (x - 2)

E. x (x - 1) (x - 5)


EXCEPT

B. x (x - 1) (x + 2)

It's not necessary for the three integers to be always consecutive for their product to be divisible by 3.


Thanks for the detailed explaination man. U rock.

Junior | Next Rank: 30 Posts
Posts: 12
Joined: Sat May 19, 2007 8:59 am
Thanked: 1 times

by samirnajeeb » Wed Jan 05, 2011 12:35 am
a. -4
b. -2 [2 more than A]
c. -1 [3 more than A]
d. 2 [6 more than A]
e. 5 [9 more than A]

nice, so we do have a pattern ... 4 answers have a difference of a multiple of 3 except B ... 3, 6, 9 are all multiples of 3

so we can select B without solving much